Question

Calendar x Course Home WebWork Math 1417HBG520 X G 11 59 edt to ist - Google Search X + bg.psu.edu/webwork2/Math-141-7HBG-S20

Homework 5: Problem 6 Previous Problem List Next (1 point) The three series A, B, and have terms An - 1110 Bn - 123 Cn = 1/2

* Calendar x Course Home X Web Work Math-141 THBG 520 X G 11 59 edt toist - Google Search X + bg.psu.edu/webwork/Math-141-7HB

0 0
Add a comment Improve this question Transcribed image text
Answer #1


Problem 7. in is divergent. But the minn) argument is not valid. for example but is convergent. Proof of divergence of antronA. since a h Comparison test Answer, c it sm) a n then s(m)o tornsill and { this convergent, by is convergent. 5. since sinnprobleme 1. snt mlo Now 148 n tanto Let an and bra 448n - an +8 Now an _ 748 n + 125 +8 zno+ns - 748 + i naturen 748 as nyo

Add a comment
Know the answer?
Add Answer to:
Calendar x Course Home WebWork Math 1417HBG520 X G 11 59 edt to ist - Google...
Your Answer:

Post as a guest

Your Name:

What's your source?

Earn Coins

Coins can be redeemed for fabulous gifts.

Not the answer you're looking for? Ask your own homework help question. Our experts will answer your question WITHIN MINUTES for Free.
Similar Homework Help Questions
  • The three series Σ Α Σ Β, and ΣC, have terms 1 1 An B =...

    The three series Σ Α Σ Β, and ΣC, have terms 1 1 An B = CHE n8 n Use the Limit Comparison Test to compare the following series to any of the above series. For each of the series below, you must enter two letters. The first is the letter (A,B, or C) of the series above that it can be legally compared to with the Limit Comparison Test. The second is C if the given series converges, or...

  • The three series A,, > Bn, and Chave terms 1 В, — 1 C - 1 А, %3 n Use the Limit Comparison Test to compare the follo...

    The three series A,, > Bn, and Chave terms 1 В, — 1 C - 1 А, %3 n Use the Limit Comparison Test to compare the following series to any of the above series. For each of the series below, you must enter two letters. The first is the letter (A,B, or C) of the series above that it can be legally compared to with the Limit Comparison Test. The second is C if the given series converges, or...

  • At least one of the answers above is NOT correct. (1 point) Each of the following statements is an attempt to show that a given series is convergent or divergent using the Comparison Test (NOT the Li...

    At least one of the answers above is NOT correct. (1 point) Each of the following statements is an attempt to show that a given series is convergent or divergent using the Comparison Test (NOT the Limit Comparison Test.) For each statement, enter C (for "correct") if the argument is valid, or enter I (for "incorrect") if any part of the argument is flawed. (Note: if the conclusion is true but the argument that led to it was wrong, you...

  • (1 point) The three series ^A,, ^ Bn, and > Cn have terms 1 An n 1 В, %3 1 С, —- = Use the Limit Comparison Test to...

    (1 point) The three series ^A,, ^ Bn, and > Cn have terms 1 An n 1 В, %3 1 С, —- = Use the Limit Comparison Test to compare the following series to any of the above series. For each of the series below, you must enter two letters. The first is the letter (A,B, or C) of the series above that it can be legally compared to with the Limit Comparison Test. The second is C if the...

  • (1 point) Each of the following statements is an attempt to show that a given series is convergen...

    (1 point) Each of the following statements is an attempt to show that a given series is convergent or divergent not using the Comparison Test (NOT the Limit Comparison Test.) For each statement, enter C (or "correct") if the argument is valid, or enterI (for "incorrect") if any part of the argument is flawed (Note: if the conclusion is true but the argument that led to it was wrong, you must enter I.) 1. For all n^ 1 arctan(n 2....

  • (1 point) Each of the following statements is an attempt to show that a given series...

    (1 point) Each of the following statements is an attempt to show that a given series is convergent or divergent using the Comparison Test (NOT the Limit Comparison Test). For each statement, enter Correct if the argument is valid, or enter Incorrect if any part of the argument is flawed. (Note: if the conclusion is true but the argument that led to it was wrong, you must enter Incorrect.) In(n) 1 1 In(n) Incorrect v 1. For all n >...

  • (1 point) Each of the following statements is an attempt to show that a given series...

    (1 point) Each of the following statements is an attempt to show that a given series is convergent or divergent using the Comparison Test (NOT the Limit Comparison Test.) For each statement, enter C (for "correct") if the argument is valid, or enter 1 (for "incorrect") if any part of the argument is flawed. (Note: if the conclusion is true but the argument that led to it was wrong, you must enter I.) 1. For all n > 2, 6...

  • (1 point) The three series [ An, Bn, and Cn have terms 1 1 An =...

    (1 point) The three series [ An, Bn, and Cn have terms 1 1 An = Bn = 1 n4' Cn n6' n Use the Limit Comparison Test to compare the following series to any of the above series. For each of the series below, you must enter two letters. The first is the letter (A, B, or C) of the series above that it can be legally compared to with the Limit Comparison Test. The second is C if...

  • (1 point) Each of the following statements is an attempt to show that a given series...

    (1 point) Each of the following statements is an attempt to show that a given series is convergent or divergent using the Comparison Test (NOT the Limit Comparison Test) For each statement, enter C (for "correct") if the argument is valid, or enter I (for "incorrect") if any part of the argument is flawed. (Note: if the conclusion is true but the argument that led to it was wrong, you must enter I.) TL 1. For all n > 1....

  • Each of the following statements is an attempt to show that a given series is convergent or diver...

    Each of the following statements is an attempt to show that a given series is convergent or divergent using the Comparison Test (NOT the Limit Comparison Test.) For each statement, enter C (for "correct") if the argument is valid, or enter I (for "incorrect") if any part of the argument is flawed. (Note: if the conclusion is true but the argument that led to it was wrong, you must enter I.) 1. For all n>1, n/1−n^3<1/n^2, and the series ∑1/n^2∑1/n^2...

ADVERTISEMENT
Free Homework Help App
Download From Google Play
Scan Your Homework
to Get Instant Free Answers
Need Online Homework Help?
Ask a Question
Get Answers For Free
Most questions answered within 3 hours.
ADVERTISEMENT
ADVERTISEMENT
ADVERTISEMENT